Generalized Behrend version for Grothendieck-Lefschetz trace formulaWhat are the automorphism groups of (principally polarized) abelian varieties?In what topology DM stacks are stacksBehaviour of euler characteristics in characteristic p for finite etale coversIs there a Grothendieck-Riemann-Roch type of theorem generalizing Grothendieck's Lefschetz trace formula Differentials for algebraic stacksOn an example by Romagny about fixed point stack not commuting with coarse moduli spaceBase change for quotient stackWhat is the need for torsion in the definition of lisse sheaves?Universal homeomorphism of stacks and etale sitesHigher dimensional generalization of an identity between traces of Hecke operators and number of elliptic curves over finite fields?

Generalized Behrend version for Grothendieck-Lefschetz trace formula


What are the automorphism groups of (principally polarized) abelian varieties?In what topology DM stacks are stacksBehaviour of euler characteristics in characteristic p for finite etale coversIs there a Grothendieck-Riemann-Roch type of theorem generalizing Grothendieck's Lefschetz trace formula Differentials for algebraic stacksOn an example by Romagny about fixed point stack not commuting with coarse moduli spaceBase change for quotient stackWhat is the need for torsion in the definition of lisse sheaves?Universal homeomorphism of stacks and etale sitesHigher dimensional generalization of an identity between traces of Hecke operators and number of elliptic curves over finite fields?













4












$begingroup$


[MOVED HERE FROM MSE.]



The statement of the Grothendieck-Lefschetz fixed point theorem is well-known. For a proper algebraic variety $X$ over $mathbb F_q$,
$$#X(mathbb F_q) =sum_i (−1)^i Tr(Fr_X, H^i_c(X, mathbb Q_l)).$$
Also known is the version for general constructible l-adic sheaves $mathcal F$:
$$sum_xin X(mathbb F_q) Tr(Fr_x,mathcal F_x)=sum_i (−1)^i Tr(Fr_X, H^i_c(X, mathcal F)).$$
Thirdly, K. Behrend proved an analog for the first formula in the context of algebraic stacks (replacing the scheme $X$ by a Noetherian algebraic stack $mathcal X$).




Now my question is: is there a version of the second formula for an
algebraic stack $mathcal X$ (with nice hypotheses if necessary)?




It would seem natural, since the second formula is a generalization of the first, and the first is true in the context of algebraic stacks by Behrend's work. However, the second formula does not follow directly from the first in the case of schemes (as far as I know: I would be glad if it were true!), so I am not in the position to easily extend the proof of the second formula in the more general context of stacks.



Thank you in advance.



ADDED QUESTION: Moreover, why is the sum on the left of the second formula finite when the scheme (or stack) is not of finite type? Behrend speaks about this problem, but I do not find where he solves it, if he does.










share|cite|improve this question











$endgroup$







  • 1




    $begingroup$
    The case of general constructible coefficients is not in Behrend's original 1993 paper as you say. But it's in his 2003 book "Derived $ell$-adic categories for algebraic stacks". (Also, you don't need $X$ proper in your formula.)
    $endgroup$
    – Dan Petersen
    Jul 7 at 20:06
















4












$begingroup$


[MOVED HERE FROM MSE.]



The statement of the Grothendieck-Lefschetz fixed point theorem is well-known. For a proper algebraic variety $X$ over $mathbb F_q$,
$$#X(mathbb F_q) =sum_i (−1)^i Tr(Fr_X, H^i_c(X, mathbb Q_l)).$$
Also known is the version for general constructible l-adic sheaves $mathcal F$:
$$sum_xin X(mathbb F_q) Tr(Fr_x,mathcal F_x)=sum_i (−1)^i Tr(Fr_X, H^i_c(X, mathcal F)).$$
Thirdly, K. Behrend proved an analog for the first formula in the context of algebraic stacks (replacing the scheme $X$ by a Noetherian algebraic stack $mathcal X$).




Now my question is: is there a version of the second formula for an
algebraic stack $mathcal X$ (with nice hypotheses if necessary)?




It would seem natural, since the second formula is a generalization of the first, and the first is true in the context of algebraic stacks by Behrend's work. However, the second formula does not follow directly from the first in the case of schemes (as far as I know: I would be glad if it were true!), so I am not in the position to easily extend the proof of the second formula in the more general context of stacks.



Thank you in advance.



ADDED QUESTION: Moreover, why is the sum on the left of the second formula finite when the scheme (or stack) is not of finite type? Behrend speaks about this problem, but I do not find where he solves it, if he does.










share|cite|improve this question











$endgroup$







  • 1




    $begingroup$
    The case of general constructible coefficients is not in Behrend's original 1993 paper as you say. But it's in his 2003 book "Derived $ell$-adic categories for algebraic stacks". (Also, you don't need $X$ proper in your formula.)
    $endgroup$
    – Dan Petersen
    Jul 7 at 20:06














4












4








4





$begingroup$


[MOVED HERE FROM MSE.]



The statement of the Grothendieck-Lefschetz fixed point theorem is well-known. For a proper algebraic variety $X$ over $mathbb F_q$,
$$#X(mathbb F_q) =sum_i (−1)^i Tr(Fr_X, H^i_c(X, mathbb Q_l)).$$
Also known is the version for general constructible l-adic sheaves $mathcal F$:
$$sum_xin X(mathbb F_q) Tr(Fr_x,mathcal F_x)=sum_i (−1)^i Tr(Fr_X, H^i_c(X, mathcal F)).$$
Thirdly, K. Behrend proved an analog for the first formula in the context of algebraic stacks (replacing the scheme $X$ by a Noetherian algebraic stack $mathcal X$).




Now my question is: is there a version of the second formula for an
algebraic stack $mathcal X$ (with nice hypotheses if necessary)?




It would seem natural, since the second formula is a generalization of the first, and the first is true in the context of algebraic stacks by Behrend's work. However, the second formula does not follow directly from the first in the case of schemes (as far as I know: I would be glad if it were true!), so I am not in the position to easily extend the proof of the second formula in the more general context of stacks.



Thank you in advance.



ADDED QUESTION: Moreover, why is the sum on the left of the second formula finite when the scheme (or stack) is not of finite type? Behrend speaks about this problem, but I do not find where he solves it, if he does.










share|cite|improve this question











$endgroup$




[MOVED HERE FROM MSE.]



The statement of the Grothendieck-Lefschetz fixed point theorem is well-known. For a proper algebraic variety $X$ over $mathbb F_q$,
$$#X(mathbb F_q) =sum_i (−1)^i Tr(Fr_X, H^i_c(X, mathbb Q_l)).$$
Also known is the version for general constructible l-adic sheaves $mathcal F$:
$$sum_xin X(mathbb F_q) Tr(Fr_x,mathcal F_x)=sum_i (−1)^i Tr(Fr_X, H^i_c(X, mathcal F)).$$
Thirdly, K. Behrend proved an analog for the first formula in the context of algebraic stacks (replacing the scheme $X$ by a Noetherian algebraic stack $mathcal X$).




Now my question is: is there a version of the second formula for an
algebraic stack $mathcal X$ (with nice hypotheses if necessary)?




It would seem natural, since the second formula is a generalization of the first, and the first is true in the context of algebraic stacks by Behrend's work. However, the second formula does not follow directly from the first in the case of schemes (as far as I know: I would be glad if it were true!), so I am not in the position to easily extend the proof of the second formula in the more general context of stacks.



Thank you in advance.



ADDED QUESTION: Moreover, why is the sum on the left of the second formula finite when the scheme (or stack) is not of finite type? Behrend speaks about this problem, but I do not find where he solves it, if he does.







ag.algebraic-geometry etale-cohomology algebraic-stacks constructible-sheaves l-adic-sheaves






share|cite|improve this question















share|cite|improve this question













share|cite|improve this question




share|cite|improve this question








edited 5 hours ago







W. Rether

















asked Jul 7 at 14:06









W. RetherW. Rether

217 bronze badges




217 bronze badges







  • 1




    $begingroup$
    The case of general constructible coefficients is not in Behrend's original 1993 paper as you say. But it's in his 2003 book "Derived $ell$-adic categories for algebraic stacks". (Also, you don't need $X$ proper in your formula.)
    $endgroup$
    – Dan Petersen
    Jul 7 at 20:06













  • 1




    $begingroup$
    The case of general constructible coefficients is not in Behrend's original 1993 paper as you say. But it's in his 2003 book "Derived $ell$-adic categories for algebraic stacks". (Also, you don't need $X$ proper in your formula.)
    $endgroup$
    – Dan Petersen
    Jul 7 at 20:06








1




1




$begingroup$
The case of general constructible coefficients is not in Behrend's original 1993 paper as you say. But it's in his 2003 book "Derived $ell$-adic categories for algebraic stacks". (Also, you don't need $X$ proper in your formula.)
$endgroup$
– Dan Petersen
Jul 7 at 20:06





$begingroup$
The case of general constructible coefficients is not in Behrend's original 1993 paper as you say. But it's in his 2003 book "Derived $ell$-adic categories for algebraic stacks". (Also, you don't need $X$ proper in your formula.)
$endgroup$
– Dan Petersen
Jul 7 at 20:06











1 Answer
1






active

oldest

votes


















8












$begingroup$

This is Theorem 4.2 of Shenghao Sun's paper $L$-Series of Artin stacks over finite fields, Algebra & Number Theory 6 (2012) pp 47–122, doi:10.2140/ant.2012.6.47, arXiv:1008.3689.




Let $f:mathscr X_0tomathscr Y_0$ be a morphism of $mathbb F_q$-algebraic stacks, and let $K_0in W^-,stra_m(mathscr X_0,overlinemathbb Q_ell)$ be a convergent complex of sheaves.
Then



(i) (Finiteness) $f_!K_0$ is a convergent complex of sheaves on
$mathscr Y_0,$ and



(ii) (Trace formula) $c_v(mathscr X_0,K_0)=c_v(mathscr Y_0,f_!K_0)$
for every integer $vge1.$




Here $c_v$ is the sum of the trace of the $v$th power of Frobenius. Applying this in the case where $mathscr Y_0$ is a point gets you what you want.






share|cite|improve this answer











$endgroup$












  • $begingroup$
    To amplify on Will's answer, the published version of the paper he mentions is projecteuclid.org/euclid.ant/1513729758
    $endgroup$
    – user141204
    Jul 7 at 14:24










  • $begingroup$
    @DavidRoberts: About Lauzières' deleted answer, you wrote (and it is now deleted) that "This answer is a rep-farming exercise by a sockpuppet troll". I'm curious, how did you find that out? I am asking because there is another, legitimate user with the same name on MO, and it is known that this Lauzières person had the habit of creating multiple unregistered users on MO (the post on Dec. 19, 2011). Is the new Lauzières the same as the old Lauzières?
    $endgroup$
    – Alex M.
    Jul 12 at 7:44










  • $begingroup$
    @Alex There are comments on now deleted questions (that 10k+ rep users can still see) pointing to various patterns of behaviour involving this user. There is discussion here: meta.mathoverflow.net/questions/4200/flood-of-similar-new-users
    $endgroup$
    – David Roberts
    Jul 12 at 9:35






  • 1




    $begingroup$
    Also, the old Lauziéres account(s) had a history of good interaction, the new account suddenly started asking poorly-motivated questions along the line of the sorts of questions the known sockpuppet accounts used.
    $endgroup$
    – David Roberts
    Jul 12 at 9:40






  • 1




    $begingroup$
    @W.Rether The stacks in question have finite type, I believe, making the sum finite.
    $endgroup$
    – Will Sawin
    5 hours ago













Your Answer








StackExchange.ready(function()
var channelOptions =
tags: "".split(" "),
id: "504"
;
initTagRenderer("".split(" "), "".split(" "), channelOptions);

StackExchange.using("externalEditor", function()
// Have to fire editor after snippets, if snippets enabled
if (StackExchange.settings.snippets.snippetsEnabled)
StackExchange.using("snippets", function()
createEditor();
);

else
createEditor();

);

function createEditor()
StackExchange.prepareEditor(
heartbeatType: 'answer',
autoActivateHeartbeat: false,
convertImagesToLinks: true,
noModals: true,
showLowRepImageUploadWarning: true,
reputationToPostImages: 10,
bindNavPrevention: true,
postfix: "",
imageUploader:
brandingHtml: "Powered by u003ca class="icon-imgur-white" href="https://imgur.com/"u003eu003c/au003e",
contentPolicyHtml: "User contributions licensed under u003ca href="https://creativecommons.org/licenses/by-sa/3.0/"u003ecc by-sa 3.0 with attribution requiredu003c/au003e u003ca href="https://stackoverflow.com/legal/content-policy"u003e(content policy)u003c/au003e",
allowUrls: true
,
noCode: true, onDemand: true,
discardSelector: ".discard-answer"
,immediatelyShowMarkdownHelp:true
);



);













draft saved

draft discarded


















StackExchange.ready(
function ()
StackExchange.openid.initPostLogin('.new-post-login', 'https%3a%2f%2fmathoverflow.net%2fquestions%2f335631%2fgeneralized-behrend-version-for-grothendieck-lefschetz-trace-formula%23new-answer', 'question_page');

);

Post as a guest















Required, but never shown

























1 Answer
1






active

oldest

votes








1 Answer
1






active

oldest

votes









active

oldest

votes






active

oldest

votes









8












$begingroup$

This is Theorem 4.2 of Shenghao Sun's paper $L$-Series of Artin stacks over finite fields, Algebra & Number Theory 6 (2012) pp 47–122, doi:10.2140/ant.2012.6.47, arXiv:1008.3689.




Let $f:mathscr X_0tomathscr Y_0$ be a morphism of $mathbb F_q$-algebraic stacks, and let $K_0in W^-,stra_m(mathscr X_0,overlinemathbb Q_ell)$ be a convergent complex of sheaves.
Then



(i) (Finiteness) $f_!K_0$ is a convergent complex of sheaves on
$mathscr Y_0,$ and



(ii) (Trace formula) $c_v(mathscr X_0,K_0)=c_v(mathscr Y_0,f_!K_0)$
for every integer $vge1.$




Here $c_v$ is the sum of the trace of the $v$th power of Frobenius. Applying this in the case where $mathscr Y_0$ is a point gets you what you want.






share|cite|improve this answer











$endgroup$












  • $begingroup$
    To amplify on Will's answer, the published version of the paper he mentions is projecteuclid.org/euclid.ant/1513729758
    $endgroup$
    – user141204
    Jul 7 at 14:24










  • $begingroup$
    @DavidRoberts: About Lauzières' deleted answer, you wrote (and it is now deleted) that "This answer is a rep-farming exercise by a sockpuppet troll". I'm curious, how did you find that out? I am asking because there is another, legitimate user with the same name on MO, and it is known that this Lauzières person had the habit of creating multiple unregistered users on MO (the post on Dec. 19, 2011). Is the new Lauzières the same as the old Lauzières?
    $endgroup$
    – Alex M.
    Jul 12 at 7:44










  • $begingroup$
    @Alex There are comments on now deleted questions (that 10k+ rep users can still see) pointing to various patterns of behaviour involving this user. There is discussion here: meta.mathoverflow.net/questions/4200/flood-of-similar-new-users
    $endgroup$
    – David Roberts
    Jul 12 at 9:35






  • 1




    $begingroup$
    Also, the old Lauziéres account(s) had a history of good interaction, the new account suddenly started asking poorly-motivated questions along the line of the sorts of questions the known sockpuppet accounts used.
    $endgroup$
    – David Roberts
    Jul 12 at 9:40






  • 1




    $begingroup$
    @W.Rether The stacks in question have finite type, I believe, making the sum finite.
    $endgroup$
    – Will Sawin
    5 hours ago















8












$begingroup$

This is Theorem 4.2 of Shenghao Sun's paper $L$-Series of Artin stacks over finite fields, Algebra & Number Theory 6 (2012) pp 47–122, doi:10.2140/ant.2012.6.47, arXiv:1008.3689.




Let $f:mathscr X_0tomathscr Y_0$ be a morphism of $mathbb F_q$-algebraic stacks, and let $K_0in W^-,stra_m(mathscr X_0,overlinemathbb Q_ell)$ be a convergent complex of sheaves.
Then



(i) (Finiteness) $f_!K_0$ is a convergent complex of sheaves on
$mathscr Y_0,$ and



(ii) (Trace formula) $c_v(mathscr X_0,K_0)=c_v(mathscr Y_0,f_!K_0)$
for every integer $vge1.$




Here $c_v$ is the sum of the trace of the $v$th power of Frobenius. Applying this in the case where $mathscr Y_0$ is a point gets you what you want.






share|cite|improve this answer











$endgroup$












  • $begingroup$
    To amplify on Will's answer, the published version of the paper he mentions is projecteuclid.org/euclid.ant/1513729758
    $endgroup$
    – user141204
    Jul 7 at 14:24










  • $begingroup$
    @DavidRoberts: About Lauzières' deleted answer, you wrote (and it is now deleted) that "This answer is a rep-farming exercise by a sockpuppet troll". I'm curious, how did you find that out? I am asking because there is another, legitimate user with the same name on MO, and it is known that this Lauzières person had the habit of creating multiple unregistered users on MO (the post on Dec. 19, 2011). Is the new Lauzières the same as the old Lauzières?
    $endgroup$
    – Alex M.
    Jul 12 at 7:44










  • $begingroup$
    @Alex There are comments on now deleted questions (that 10k+ rep users can still see) pointing to various patterns of behaviour involving this user. There is discussion here: meta.mathoverflow.net/questions/4200/flood-of-similar-new-users
    $endgroup$
    – David Roberts
    Jul 12 at 9:35






  • 1




    $begingroup$
    Also, the old Lauziéres account(s) had a history of good interaction, the new account suddenly started asking poorly-motivated questions along the line of the sorts of questions the known sockpuppet accounts used.
    $endgroup$
    – David Roberts
    Jul 12 at 9:40






  • 1




    $begingroup$
    @W.Rether The stacks in question have finite type, I believe, making the sum finite.
    $endgroup$
    – Will Sawin
    5 hours ago













8












8








8





$begingroup$

This is Theorem 4.2 of Shenghao Sun's paper $L$-Series of Artin stacks over finite fields, Algebra & Number Theory 6 (2012) pp 47–122, doi:10.2140/ant.2012.6.47, arXiv:1008.3689.




Let $f:mathscr X_0tomathscr Y_0$ be a morphism of $mathbb F_q$-algebraic stacks, and let $K_0in W^-,stra_m(mathscr X_0,overlinemathbb Q_ell)$ be a convergent complex of sheaves.
Then



(i) (Finiteness) $f_!K_0$ is a convergent complex of sheaves on
$mathscr Y_0,$ and



(ii) (Trace formula) $c_v(mathscr X_0,K_0)=c_v(mathscr Y_0,f_!K_0)$
for every integer $vge1.$




Here $c_v$ is the sum of the trace of the $v$th power of Frobenius. Applying this in the case where $mathscr Y_0$ is a point gets you what you want.






share|cite|improve this answer











$endgroup$



This is Theorem 4.2 of Shenghao Sun's paper $L$-Series of Artin stacks over finite fields, Algebra & Number Theory 6 (2012) pp 47–122, doi:10.2140/ant.2012.6.47, arXiv:1008.3689.




Let $f:mathscr X_0tomathscr Y_0$ be a morphism of $mathbb F_q$-algebraic stacks, and let $K_0in W^-,stra_m(mathscr X_0,overlinemathbb Q_ell)$ be a convergent complex of sheaves.
Then



(i) (Finiteness) $f_!K_0$ is a convergent complex of sheaves on
$mathscr Y_0,$ and



(ii) (Trace formula) $c_v(mathscr X_0,K_0)=c_v(mathscr Y_0,f_!K_0)$
for every integer $vge1.$




Here $c_v$ is the sum of the trace of the $v$th power of Frobenius. Applying this in the case where $mathscr Y_0$ is a point gets you what you want.







share|cite|improve this answer














share|cite|improve this answer



share|cite|improve this answer








edited Jul 12 at 4:05









David Roberts

18.2k4 gold badges64 silver badges188 bronze badges




18.2k4 gold badges64 silver badges188 bronze badges










answered Jul 7 at 14:21









Will SawinWill Sawin

71.2k7 gold badges144 silver badges296 bronze badges




71.2k7 gold badges144 silver badges296 bronze badges











  • $begingroup$
    To amplify on Will's answer, the published version of the paper he mentions is projecteuclid.org/euclid.ant/1513729758
    $endgroup$
    – user141204
    Jul 7 at 14:24










  • $begingroup$
    @DavidRoberts: About Lauzières' deleted answer, you wrote (and it is now deleted) that "This answer is a rep-farming exercise by a sockpuppet troll". I'm curious, how did you find that out? I am asking because there is another, legitimate user with the same name on MO, and it is known that this Lauzières person had the habit of creating multiple unregistered users on MO (the post on Dec. 19, 2011). Is the new Lauzières the same as the old Lauzières?
    $endgroup$
    – Alex M.
    Jul 12 at 7:44










  • $begingroup$
    @Alex There are comments on now deleted questions (that 10k+ rep users can still see) pointing to various patterns of behaviour involving this user. There is discussion here: meta.mathoverflow.net/questions/4200/flood-of-similar-new-users
    $endgroup$
    – David Roberts
    Jul 12 at 9:35






  • 1




    $begingroup$
    Also, the old Lauziéres account(s) had a history of good interaction, the new account suddenly started asking poorly-motivated questions along the line of the sorts of questions the known sockpuppet accounts used.
    $endgroup$
    – David Roberts
    Jul 12 at 9:40






  • 1




    $begingroup$
    @W.Rether The stacks in question have finite type, I believe, making the sum finite.
    $endgroup$
    – Will Sawin
    5 hours ago
















  • $begingroup$
    To amplify on Will's answer, the published version of the paper he mentions is projecteuclid.org/euclid.ant/1513729758
    $endgroup$
    – user141204
    Jul 7 at 14:24










  • $begingroup$
    @DavidRoberts: About Lauzières' deleted answer, you wrote (and it is now deleted) that "This answer is a rep-farming exercise by a sockpuppet troll". I'm curious, how did you find that out? I am asking because there is another, legitimate user with the same name on MO, and it is known that this Lauzières person had the habit of creating multiple unregistered users on MO (the post on Dec. 19, 2011). Is the new Lauzières the same as the old Lauzières?
    $endgroup$
    – Alex M.
    Jul 12 at 7:44










  • $begingroup$
    @Alex There are comments on now deleted questions (that 10k+ rep users can still see) pointing to various patterns of behaviour involving this user. There is discussion here: meta.mathoverflow.net/questions/4200/flood-of-similar-new-users
    $endgroup$
    – David Roberts
    Jul 12 at 9:35






  • 1




    $begingroup$
    Also, the old Lauziéres account(s) had a history of good interaction, the new account suddenly started asking poorly-motivated questions along the line of the sorts of questions the known sockpuppet accounts used.
    $endgroup$
    – David Roberts
    Jul 12 at 9:40






  • 1




    $begingroup$
    @W.Rether The stacks in question have finite type, I believe, making the sum finite.
    $endgroup$
    – Will Sawin
    5 hours ago















$begingroup$
To amplify on Will's answer, the published version of the paper he mentions is projecteuclid.org/euclid.ant/1513729758
$endgroup$
– user141204
Jul 7 at 14:24




$begingroup$
To amplify on Will's answer, the published version of the paper he mentions is projecteuclid.org/euclid.ant/1513729758
$endgroup$
– user141204
Jul 7 at 14:24












$begingroup$
@DavidRoberts: About Lauzières' deleted answer, you wrote (and it is now deleted) that "This answer is a rep-farming exercise by a sockpuppet troll". I'm curious, how did you find that out? I am asking because there is another, legitimate user with the same name on MO, and it is known that this Lauzières person had the habit of creating multiple unregistered users on MO (the post on Dec. 19, 2011). Is the new Lauzières the same as the old Lauzières?
$endgroup$
– Alex M.
Jul 12 at 7:44




$begingroup$
@DavidRoberts: About Lauzières' deleted answer, you wrote (and it is now deleted) that "This answer is a rep-farming exercise by a sockpuppet troll". I'm curious, how did you find that out? I am asking because there is another, legitimate user with the same name on MO, and it is known that this Lauzières person had the habit of creating multiple unregistered users on MO (the post on Dec. 19, 2011). Is the new Lauzières the same as the old Lauzières?
$endgroup$
– Alex M.
Jul 12 at 7:44












$begingroup$
@Alex There are comments on now deleted questions (that 10k+ rep users can still see) pointing to various patterns of behaviour involving this user. There is discussion here: meta.mathoverflow.net/questions/4200/flood-of-similar-new-users
$endgroup$
– David Roberts
Jul 12 at 9:35




$begingroup$
@Alex There are comments on now deleted questions (that 10k+ rep users can still see) pointing to various patterns of behaviour involving this user. There is discussion here: meta.mathoverflow.net/questions/4200/flood-of-similar-new-users
$endgroup$
– David Roberts
Jul 12 at 9:35




1




1




$begingroup$
Also, the old Lauziéres account(s) had a history of good interaction, the new account suddenly started asking poorly-motivated questions along the line of the sorts of questions the known sockpuppet accounts used.
$endgroup$
– David Roberts
Jul 12 at 9:40




$begingroup$
Also, the old Lauziéres account(s) had a history of good interaction, the new account suddenly started asking poorly-motivated questions along the line of the sorts of questions the known sockpuppet accounts used.
$endgroup$
– David Roberts
Jul 12 at 9:40




1




1




$begingroup$
@W.Rether The stacks in question have finite type, I believe, making the sum finite.
$endgroup$
– Will Sawin
5 hours ago




$begingroup$
@W.Rether The stacks in question have finite type, I believe, making the sum finite.
$endgroup$
– Will Sawin
5 hours ago

















draft saved

draft discarded
















































Thanks for contributing an answer to MathOverflow!


  • Please be sure to answer the question. Provide details and share your research!

But avoid


  • Asking for help, clarification, or responding to other answers.

  • Making statements based on opinion; back them up with references or personal experience.

Use MathJax to format equations. MathJax reference.


To learn more, see our tips on writing great answers.




draft saved


draft discarded














StackExchange.ready(
function ()
StackExchange.openid.initPostLogin('.new-post-login', 'https%3a%2f%2fmathoverflow.net%2fquestions%2f335631%2fgeneralized-behrend-version-for-grothendieck-lefschetz-trace-formula%23new-answer', 'question_page');

);

Post as a guest















Required, but never shown





















































Required, but never shown














Required, but never shown












Required, but never shown







Required, but never shown

































Required, but never shown














Required, but never shown












Required, but never shown







Required, but never shown







Popular posts from this blog

Get product attribute by attribute group code in magento 2get product attribute by product attribute group in magento 2Magento 2 Log Bundle Product Data in List Page?How to get all product attribute of a attribute group of Default attribute set?Magento 2.1 Create a filter in the product grid by new attributeMagento 2 : Get Product Attribute values By GroupMagento 2 How to get all existing values for one attributeMagento 2 get custom attribute of a single product inside a pluginMagento 2.3 How to get all the Multi Source Inventory (MSI) locations collection in custom module?Magento2: how to develop rest API to get new productsGet product attribute by attribute group code ( [attribute_group_code] ) in magento 2

Category:9 (number) SubcategoriesMedia in category "9 (number)"Navigation menuUpload mediaGND ID: 4485639-8Library of Congress authority ID: sh85091979ReasonatorScholiaStatistics

Magento 2.3: How do i solve this, Not registered handle, on custom form?How can i rewrite TierPrice Block in Magento2magento 2 captcha not rendering if I override layout xmlmain.CRITICAL: Plugin class doesn't existMagento 2 : Problem while adding custom button order view page?Magento 2.2.5: Overriding Admin Controller sales/orderMagento 2.2.5: Add, Update and Delete existing products Custom OptionsMagento 2.3 : File Upload issue in UI Component FormMagento2 Not registered handleHow to configured Form Builder Js in my custom magento 2.3.0 module?Magento 2.3. How to create image upload field in an admin form